LSAT and Law School Admissions Forum

Get expert LSAT preparation and law school admissions advice from PowerScore Test Preparation.

 1800-HELPME
  • Posts: 16
  • Joined: May 19, 2017
|
#36980
Hi,

I'm having trouble locating the evidence for (B). Where in the passage does it suggest that Garber would agree that the owners of high-priced original tulip bulbs could have expected to at least recoup their original investments from the sales of the many bulbs propagated from the original bulbs?
User avatar
 Jonathan Evans
PowerScore Staff
  • PowerScore Staff
  • Posts: 726
  • Joined: Jun 09, 2016
|
#37098
Hi, 800,

Good question! In Garber's critique of Mackay's view, he suggests that "an original bulb can generate a reasonable return on investment even if the price of descendent bulbs decreases dramatically." In this line (55) and the preceding lines (ll 48-55) we have solid evidence that in Garber's description of Dutch tulip market "fundamentals," the owners and propagators of the initial rare tulip bulbs made a rational economic decision and were not necessarily the victims of a speculative bubble.

Note in addition the manner in which the writers of the LSAT have qualified the statement in answer choice (B). To wit "many of the owners [...] could have expected to at least recoup..."

This question tests your understanding of one of the viewpoints in this passage, that of Garber. Answer Choice (B) is directly consistent with Garber's view.

To make an effective prephrase for this question, you could start by noting that the question is asking for what Garber "would be most likely to agree with."
  1. Start by recapitulating for yourself Garber's perspective.
  2. Make note of where Garber's viewpoint is discussed.
  3. Eliminate answers that don't pass the smell test (they don't match with Garber's viewpoint).
  4. Verify that there is adequate evidence in the passage to support one of the remaining answer choices (in this case ll 48-55).
Good question! I hope this helps!
 brcibake
  • Posts: 55
  • Joined: Jul 19, 2017
|
#39553
Why couldn't D be correct? To me, the passage suggests that the people who bought the dutch tulip bulbs were rational.
Thank you
 nicholaspavic
PowerScore Staff
  • PowerScore Staff
  • Posts: 271
  • Joined: Jun 12, 2017
|
#39597
Hi brci,

Answer Option (D) goes well beyond what Garber's stated position was in the passage. We have no idea of what Garber's broader views are about this topic, or if Garber thought that Dutch tulip bulb investors "were generally rational in all their investments." That is an Exaggerated Answer that cannot pass our Fact Test because it is a bold statement that is not traceable to any portion of the passage. Remember that for Must Be True questions like this one, support must always be found directly in some portion of our passage.

Thanks for the great question!
 LSAT2018
  • Posts: 242
  • Joined: Jan 10, 2018
|
#58088
I understand why answer (B) is the correct choice, but was wondering how the other answers would be eliminated. Answer choices (C), (D), and (E) all relate to whether the market was rational/irrational, and the passage reads, 'But this does not mean that the high prices of original bulbs are irrational, for earnings derivable from the millions of bulbs descendent from the original bulbs can be very high, even if each individual descendent bulb commands a very low price' (lines 46-52).

And from the other information about market fundamentals, is it possible to infer that because Garber believes the market to be rational, there was no speculative bubble? So Answer (C) is a Mistaken Reversal of this, and thus can be eliminated. Answer (D) refers to the general investment behavior of most people who invested in Dutch tulip bulbs, when in fact, the passage only refers to one instance when they were rational. As for Answer (E), I would like to ask whether it can be determined/inferred whether Mackay refers to the concept of rationality in the first place. Otherwise, how would (E) be eliminated?
 jwheeler
  • Posts: 39
  • Joined: Aug 19, 2018
|
#60210
I chose E, but now I can't find support in the passage for Mackay saying that the low price was "irrational". I think I made that up in my mind as I was going. Since it's not from the passage and we don't necessarily know what he thinks about the rationality of the low prices, is that the basis for eliminating E as a contender?
 James Finch
PowerScore Staff
  • PowerScore Staff
  • Posts: 943
  • Joined: Sep 06, 2017
|
#61777
Hi J Wheeler,

Absolutely that is one reason for eliminating this answer choice. When reading through passages, try and mentally note both what is being explicitly said and what isn't--how much do we really know about the subject? Here, though, I think this is more of an opposite answer, which may be what led you to choose it (the right elements being in the wrong places). Everything we're given about Mackay's account would indicate that, as a speculative bubble, he believed that the high prices that the bulbs initially sold at were irrational, but not necessarily the much lower ones that were seen after the bottom fell out of the market. So the proper inference would be that Garber believes Mackay mistakenly thought the high prices were irrational, not the lower ones.

Hope this clears things up!
User avatar
 annabelle.swift
  • Posts: 54
  • Joined: Sep 01, 2021
|
#94604
Hi, I understand that B is connected to the sentence "an original bulb can generate a reasonable return on investment even if the price of descendent bulbs decreases dramatically."

However, I initially eliminated B because of the word "can."

This was my thought process to eliminate B: Just because an original bulb CAN generate a reasonable ROI does not mean that MANY owners of original bulbs could have EXPECTED to at least recoup their investment. Just because something is possible does not mean you should expect it to happen.

Is my reasoning wrong? Or is B just the best answer despite the problem I think I see in it?
 Adam Tyson
PowerScore Staff
  • PowerScore Staff
  • Posts: 5153
  • Joined: Apr 14, 2011
|
#94608
I think we have to look at two more bits of information, annabelle: the use of "could" in answer B, matching the strength of "can" in the passage, and the fact that Garber referred to this pricing pattern as "standard" (and the author seems to favor Garber's analysis). Thus, it's not just something that can happen once in a while; it's something that would be generally expected. It's fundamental in the tulip bulb market. It's the standard.

And ultimately, yes, this answer is better than the others despite any concerns we might have about "many." We aren't looking for proof that many people did have that expectation, or even that it was reasonable for them to have that expectation. We just need evidence that the author would agree that many speculators COULD have had that expectation. Since that is the standard pricing pattern, and long-term earnings can be very high despite dropping prices, we have more than enough support for answer B.
 ltowns1
  • Posts: 61
  • Joined: May 16, 2017
|
#98611
Can’t you also eliminate (C) on the idea that Garber is not really talking about rational/irrationality of the market in general, he only mentioned rationality as it relates to the original bulbs and not the other descendant bulbs?

Get the most out of your LSAT Prep Plus subscription.

Analyze and track your performance with our Testing and Analytics Package.